Mathcenter Forum  

Go Back   Mathcenter Forum > ค้นหาในห้อง
สมัครสมาชิก คู่มือการใช้ รายชื่อสมาชิก ปฏิทิน ข้อความวันนี้

แสดงผลลัพธ์ตั้งแต่ 1 ถึง 8 จากทั้งหมด 8
ใช้เวลาค้นหา 0.00 วินาที.
ค้นหา: ข้อความของคุณ: gools
ห้อง: อสมการ 19 พฤศจิกายน 2007, 23:44
คำตอบ: 155
ปักหมุด: Inequality Marathon
เปิดอ่าน: 89,844
ข้อความของคุณ gools
แต่ $a(a-b)(a-c),b(b-c)(b-a),c(c-a)(c-b)$...

แต่ $a(a-b)(a-c),b(b-c)(b-a),c(c-a)(c-b)$ ไม่เป็นจำนวนจริงบวกทั้งหมดนะครับ
ห้อง: อสมการ 16 กันยายน 2007, 02:37
คำตอบ: 155
ปักหมุด: Inequality Marathon
เปิดอ่าน: 89,844
ข้อความของคุณ gools
น่าจะอ้างอิงด้วยนะครับว่ามาจากไหน :D

น่าจะอ้างอิงด้วยนะครับว่ามาจากไหน :D
ห้อง: อสมการ 05 มกราคม 2006, 23:22
คำตอบ: 155
ปักหมุด: Inequality Marathon
เปิดอ่าน: 89,844
ข้อความของคุณ gools
Post แทน $x=a^2,y=b^2,z=c^2$ ดังนั้นคำถามใหม่คือ ให้...

แทน $x=a^2,y=b^2,z=c^2$ ดังนั้นคำถามใหม่คือ
ให้ $a , b , c$ เป็นจำนวนจริงบวกที่ $a^2+b^2+c^2=3$
จงพิสูจน์ว่า $a+b+c \geq a^2b^2+b^2c^2+c^2a^2$

เปลี่ยนรูปใหม่
\[\begin{array}{rcl}a+b+c &\geq&...
ห้อง: อสมการ 05 มกราคม 2006, 15:26
คำตอบ: 155
ปักหมุด: Inequality Marathon
เปิดอ่าน: 89,844
ข้อความของคุณ gools
Post ข้อ 1 ยังทำไม่ได้เลยครับ...

ข้อ 1 ยังทำไม่ได้เลยครับ เหลือแค่วิธีเดียวแล้วคือถึกเอาแต่ยังไม่ได้ลอง

ส่วนข้อ 2 ครับ จากเงื่อนไขและ AM-HM จะได้ว่า
\[3 \geq \frac{1}{x}+\frac{1}{y}+\frac{1}{z} \geq...
ห้อง: อสมการ 06 สิงหาคม 2005, 00:20
คำตอบ: 155
ปักหมุด: Inequality Marathon
เปิดอ่าน: 89,844
ข้อความของคุณ gools
Post อสมการสุดท้ายตรงกับ (2(a+b+c)+3)(a+b+c-3) \geq 0...

อสมการสุดท้ายตรงกับ (2(a+b+c)+3)(a+b+c-3) \geq 0 ครับ
ห้อง: อสมการ 30 กรกฎาคม 2005, 17:36
คำตอบ: 155
ปักหมุด: Inequality Marathon
เปิดอ่าน: 89,844
ข้อความของคุณ gools
Post มาจาก a+b+c=ab+bc+ca \leq \frac{(a+b+c)^2}{3} ครับ

มาจาก
a+b+c=ab+bc+ca \leq \frac{(a+b+c)^2}{3}
ครับ
ห้อง: อสมการ 30 กรกฎาคม 2005, 17:30
คำตอบ: 155
ปักหมุด: Inequality Marathon
เปิดอ่าน: 89,844
ข้อความของคุณ gools
Post จาก Cauchy เราได้ว่า \sum...

จาก Cauchy เราได้ว่า
\sum \frac{a^2+2bc}{a^3+b^2+c^2} \leq \sum \frac{a^2+b^2+c^2}{a^3+b^2+c^2}=(a^2+b^2+c^2)\sum\frac{1}{a^3+b^2+c^2}\leq (a^2+b^2+c^2)\sum\frac{a+c^2+c^2}{(a^2+b^2+c^2)^2}=\frac{...
ห้อง: อสมการ 28 กรกฎาคม 2005, 16:24
คำตอบ: 155
ปักหมุด: Inequality Marathon
เปิดอ่าน: 89,844
ข้อความของคุณ gools
Post ข้อแรกนะครับ ให้ a=10^{25}...

ข้อแรกนะครับ
ให้ a=10^{25} ดังนั้นเราต้องเปรียบเทียบระหว่าง \frac{a+3}{a^2+5a+7} กับ \frac{a+5}{a^2+9a+21}
ลองคูณไขว้ดู จะพบว่า (a+3)(a^2+9a+21) > (a+5)(a^2+5a+7)
ดังนั้น \frac{a+3}{a^2+5a+7} >...
แสดงผลลัพธ์ตั้งแต่ 1 ถึง 8 จากทั้งหมด 8

 
ทางลัดสู่ห้อง

เวลาที่แสดงทั้งหมด เป็นเวลาที่ประเทศไทย (GMT +7) ขณะนี้เป็นเวลา 20:47


Powered by vBulletin® Copyright ©2000 - 2024, Jelsoft Enterprises Ltd.
Modified by Jetsada Karnpracha